3
$\begingroup$

Let $(X,\leq)$ be a poset, $X=\{x_1,x_2,\dots,x_n\}$. Preference matrix $\textbf{P}=[p_{ij}]$ (which is known and fixed), satisfies $p_{ij}=1-p_{ji},p_{ii}=\frac{1}{2}$, and

$$\forall i \neq j, x_i \leq x_j \iff p_{ij} < \frac{1}{2},$$

$$ \forall i \neq j, x_i \| x_j \;(x_i \text{ can't compare with } x_j) \Longleftrightarrow p_{ij}=\frac{1}{2}.$$

Define sigmod function $f(x)=\frac{1}{1+e^{-x}}$, and
$$ g(a_1,a_2,\dots,a_n) = \sum_{i,j}|f(a_i-a_j)-p_{ij}|. $$

I'd like to know a good lower bound for the function $g$. I'm sure this lower bound is greater than $0$. Can anyone give me some hint?

$\endgroup$
5
  • $\begingroup$ What do you mean by "a good lower bound"? What properties of the bound will make it good enough for you? $\endgroup$ Commented Jun 2, 2024 at 17:46
  • $\begingroup$ I think a non-trivial lower bound is sufficient , possibly related to chains and antichains. $\endgroup$ Commented Jun 3, 2024 at 3:24
  • $\begingroup$ What if $X=\{1,\dots,n\}$, $a_i=i$ for all $i$, and $p_{ij}=f(i-j)$ for all $i,j$? Then $g(a_1,\dots,a_n)=0$. $\endgroup$ Commented Jun 3, 2024 at 12:51
  • $\begingroup$ OK. This is a special case of a totally ordered set; while I'd like to consider the partially ordered set that is not totally ordered. $\endgroup$ Commented Jun 3, 2024 at 14:48
  • $\begingroup$ Saying $p_{ii} = \frac{1}{2}$ is redundant. $\endgroup$ Commented May 1 at 12:46

1 Answer 1

0
$\begingroup$

Under the conditions specified in your post, the best lower bound on $g(a_1,\dots,a_n)$ is $\sum'_{i,j}|f(a_i-a_j)-1/2|$, where $\sum'_{i,j}$ denotes the summation over all pairs $(i,j)$ with $x_i ||x_j$. Indeed, if for some $n$-tuple $(a_1,\dots,a_n)$ with $a_1<\cdots<a_n$ and all pairs $(i,j)$ with $i\ne j$ and $x_i\le x_j$ we have $p_{ij}=f(a_i,a_j)$, then your conditions on the $p_{ij}$'s hold, whereas $g(a_1,\dots,a_n)=\sum'_{i,j}|f(a_i-a_j)-1/2|$, so that the lower bound is attained.

$\endgroup$
3
  • $\begingroup$ I think there may not exist $(a_1,...,a_n)$ such that $p_{ij}=f(a_i,a_j)$ for all $x_i \leq x_j$. Because the number of equations is greater than the number of $a_i$( i.e. $n$). $\endgroup$ Commented Jun 3, 2024 at 15:44
  • 1
    $\begingroup$ @MixiAndrew : Of course, the only bound on $g(a_1,\dots,a_n)$ that is exact for all feasible $p_{ij}$'s and all $a_i$'s is the tautological bound, that is, $g(a_1,\dots,a_n)$ itself. Any non-tautological bound will be exact only under a certain, rather narrow condition. In this answer, this condition for the exactness is that the $p_{ij}$'s and $a_i$'s are matching, in the sense that $p_{ij}=f(a_i-a_j)$ for $(i,j)$ such that .... However, the mentioned lower bound holds for all $p_{ij}$'s and all $a_i$'s satisfying your conditions. $\endgroup$ Commented Jun 3, 2024 at 16:41
  • $\begingroup$ Thank you for your attention to this problem. I don't fully understand your response. I should include a constraint in my question: $p_{ij}$'s are known and fixed. $\endgroup$ Commented Jun 4, 2024 at 1:59

You must log in to answer this question.

Start asking to get answers

Find the answer to your question by asking.

Ask question

Explore related questions

See similar questions with these tags.